Difference between revisions of "2017 AMC 10B Problems/Problem 1"

m
Line 6: Line 6:
  
 
==Solution 1==
 
==Solution 1==
Just try out the answer choices. Multiplying <math>12</math> by <math>3</math> and then adding <math>11</math> and reversing the digits gives you <math>74</math>, which works, so the answer is <math>\boxed{\textbf{(B)}\ 12}</math>.
+
We try out each answer choice. Multiplying <math>12</math> by <math>3</math>, adding <math>11</math>, and reversing the digits yields <math>74</math>. Therefore the answer is <math>\boxed{\textbf{(B)}\ 12}</math>.
  
 
==Solution 2==
 
==Solution 2==
 
Working backwards, we reverse the digits of each number from <math>71</math>~<math>75</math> and subtract <math>11</math> from each, so we have
 
Working backwards, we reverse the digits of each number from <math>71</math>~<math>75</math> and subtract <math>11</math> from each, so we have
 
<cmath>6, 16, 26, 36, 46</cmath>
 
<cmath>6, 16, 26, 36, 46</cmath>
The only numbers from this list that are divisible by <math>3</math> are <math>6</math> and <math>36</math>. We divide both by <math>3</math>, yielding <math>2</math> and <math>12</math>. Since <math>2</math> is not among the answer choices, the correct answer would be <math>\boxed{\textbf{(B)}\ 12}</math>.
+
The only numbers from this list that are divisible by <math>3</math> are <math>6</math> and <math>36</math>. We divide both by <math>3</math>, yielding <math>2</math> and <math>12</math>. Since <math>2</math> is not a two-digit number, the answer is <math>\boxed{\textbf{(B)}\ 12}</math>.
  
 
==See Also==
 
==See Also==
 
{{AMC10 box|year=2017|ab=B|before=First Problem|num-a=2}}
 
{{AMC10 box|year=2017|ab=B|before=First Problem|num-a=2}}
 
{{MAA Notice}}
 
{{MAA Notice}}

Revision as of 13:44, 16 February 2017

Problem

Mary thought of a positive two-digit number. She multiplied it by $3$ and added $11$. Then she switched the digits of the result, obtaining a number between $71$ and $75$, inclusive. What was Mary's number?

$\textbf{(A)}\ 11\qquad\textbf{(B)}\ 12\qquad\textbf{(C)}\ 13\qquad\textbf{(D)}\ 14\qquad\textbf{(E)}\ 15$

Solution 1

We try out each answer choice. Multiplying $12$ by $3$, adding $11$, and reversing the digits yields $74$. Therefore the answer is $\boxed{\textbf{(B)}\ 12}$.

Solution 2

Working backwards, we reverse the digits of each number from $71$~$75$ and subtract $11$ from each, so we have \[6, 16, 26, 36, 46\] The only numbers from this list that are divisible by $3$ are $6$ and $36$. We divide both by $3$, yielding $2$ and $12$. Since $2$ is not a two-digit number, the answer is $\boxed{\textbf{(B)}\ 12}$.

See Also

2017 AMC 10B (ProblemsAnswer KeyResources)
Preceded by
First Problem
Followed by
Problem 2
1 2 3 4 5 6 7 8 9 10 11 12 13 14 15 16 17 18 19 20 21 22 23 24 25
All AMC 10 Problems and Solutions

The problems on this page are copyrighted by the Mathematical Association of America's American Mathematics Competitions. AMC logo.png